Difference between revisions of "1996 AHSME Problems/Problem 1"

m (Problem)
m
 
Line 11: Line 11:
 
Doing the addition as is, we get <math>641 + 852 + 973 = 2466</math>.  This number is <math>10</math> larger than the desired sum of <math>2456</math>.  Therefore, we must make one of the three numbers <math>10</math> smaller.
 
Doing the addition as is, we get <math>641 + 852 + 973 = 2466</math>.  This number is <math>10</math> larger than the desired sum of <math>2456</math>.  Therefore, we must make one of the three numbers <math>10</math> smaller.
  
We may either change <math>641 \rightarrow 631</math>, <math>852 \rightarrow 842</math>, or <math>973 \rightarrow 963</math>.  Either change results in a valid sum.  The largest digit that could be changed is thus the <math>7</math> in the number <math>973</math>, and the answer is <math>\boxed{D}</math>.
+
We may either change <math>641 \rightarrow 631</math>, <math>852 \rightarrow 842</math>, or <math>973 \rightarrow 963</math>.  Either change results in a valid sum.  The largest digit that could be changed is thus the <math>7</math> in the number <math>973</math>, and the answer is <math>\boxed{\textbf{(D) }7}</math>.
  
 
==See also==
 
==See also==
 
{{AHSME box|year=1996|before=First question|num-a=2}}
 
{{AHSME box|year=1996|before=First question|num-a=2}}
 
{{MAA Notice}}
 
{{MAA Notice}}

Latest revision as of 16:06, 14 July 2021

Problem

The addition below is incorrect. What is the largest digit that can be changed to make the addition correct?

$\begin{tabular}{rr}&\ \texttt{6 4 1}\\ &\texttt{8 5 2}\\ &+\texttt{9 7 3}\\ \hline  &\texttt{2 4 5 6}\end{tabular}$

$\text{(A)}\ 4\qquad\text{(B)}\ 5\qquad\text{(C)}\ 6\qquad\text{(D)}\ 7\qquad\text{(E)}\ 8$

Solution

Doing the addition as is, we get $641 + 852 + 973 = 2466$. This number is $10$ larger than the desired sum of $2456$. Therefore, we must make one of the three numbers $10$ smaller.

We may either change $641 \rightarrow 631$, $852 \rightarrow 842$, or $973 \rightarrow 963$. Either change results in a valid sum. The largest digit that could be changed is thus the $7$ in the number $973$, and the answer is $\boxed{\textbf{(D) }7}$.

See also

1996 AHSME (ProblemsAnswer KeyResources)
Preceded by
First question
Followed by
Problem 2
1 2 3 4 5 6 7 8 9 10 11 12 13 14 15 16 17 18 19 20 21 22 23 24 25 26 27 28 29 30
All AHSME Problems and Solutions

The problems on this page are copyrighted by the Mathematical Association of America's American Mathematics Competitions. AMC logo.png